*NURSING > EXAM > RNSG 2331 - Exam 2-Capstone-1. (100% correct answers plus rationale) (All)

RNSG 2331 - Exam 2-Capstone-1. (100% correct answers plus rationale)

Document Content and Description Below

RNSG 2331 - Exam 2-Capstone-1 Question: The nurse makes selections from the hospital menu for a client who is confused and suspicious of others. Which menu choice is best? You answered this que... stion Correctly 1. Ham and vegetable casserole 2. Cheese and crackers 3. Caffeine free tea 4. Correct: A client who is suspicious of others needs foods that are packaged and can see them opened. 1. Incorrect: A client who is suspicious of others needs to be able to identify the ingredients in the food that is being eaten. A casserole contains many ingredients and the client may fear that something has been added to the food. 2. Incorrect: Finger foods are best for clients that are manic. 3. Incorrect: Drinks and foods with no caffeine are okay for the confused and suspicious client but this menu choice is not the best choice from the list here. Question: Two days after a myocardial infarction, a client begins reporting orthopnea and dyspnea. Further assessment reveals bi-basilar crackles, jugular venous distension, an S3 heart sound, a BP of 100/60 mm Hg, and apical pulse of 90 beats per minute. The urine output has steadily declined over the past 12 hours. What should the nurse do first? You answered this question Correctly 1. Notify the primary healthcare provider. 2. Increase the IV rate. 4. Observe for cardiac arrhythmias. 3. Correct: Elevate the head of the bed first. The client is reporting inability to breathe. (Orthopnea means the client needs to sit up to breathe better.) With ANY client having difficulty breathing, the first intervention for the nurse is to sit the client up. This client is showing s/s of heart failure. 1. Incorrect: Your next step is to call the primary healthcare provider after you do something to try to fix the problem. 2. Incorrect: Increasing the IV rate is contraindicated and would make the problem worse. 4. Incorrect: After an MI, all clients are observed for cardiac arrhythmias. This, however, does not fix the problem. Question: Which menu selection by the client diagnosed with nephrotic syndrome indicates that teaching of proper diet was understood? You answered this question Correctly 1. Pancakes with whipped butter, syrup, bacon, apple juice 3. Grits, fresh fruit, toast, coffee 4. Bagel with jelly, hash browns, tea 2. Correct: Client needs low sodium and increased proteins. 1. Incorrect: This selection is too high in sodium and fats. 3. Incorrect: This selection has no protein. Remember, nephrotic syndrome is the exception to the rule of limiting protein. These clients need increased protein to compensate for the large loss of protein in the urine. 4. Incorrect: This selection has no protein. Remember, nephrotic syndrome is the exception to the rule of limiting protein. These clients need increased protein to compensate for the large loss of protein in the urine. Question: The nurse assesses a multigravida who is four hours postpartum. Findings include that fundus is firm, 1 centimeter above the umbilicus, and deviated to the right side. The lochia is moderately heavy and bright red. Which nursing intervention has priority? You answered this question Correctly 1. Massage the fundus. 2. Administer intravenous oxytocin. 3. Document these normal findings. 4. Correct: These findings are caused by a full bladder, which prevents the uterus from contracting down and achieving homeostasis. Once the bladder is empty, the fundus will contract adequately and return to its normal location at level of umbilicus or 1 finger breadth below the umbilicus and in the midline. A distended bladder will displace the uterus, usually to the right. 1. Incorrect: The nurse may check fundus after client voids to ensure that this fixes the problem. 2. Incorrect: Administering oxytocin is not the first intervention for this issue. 3. Incorrect: These are not normal findings so this would be incorrect information for the nurse to document. Question: What risk factors should the nurse include when conducting a class about type 2 diabetes mellitus? You answered this question Correctly 2. Being underweight. 3. Having type 1 diabetes as a child increases risk for type 2 diabetes. 4. Caucasians are more likely to develop type 2 diabetes than Hispanics. 1. , & 5. Correct: If the body stores fat primarily in the abdomen, risk of type 2 diabetes is greater than if body stores fat elsewhere, such as hips and thighs. Women with polycystic ovary syndrome have increased risk of diabetes. 2. Incorrect: Being overweight is a primary risk factor for type 2 diabetes. The more fatty tissue, the more resistant cells become to insulin. 3. Incorrect: A type 1 diabetic will remain a type 1 diabetic. 4. Incorrect: Blacks, Hispanics, American Indians, and Asian Americans are more likely to develop type 2 diabetes than Caucasians are. Question: What should the nurse include when providing education to a client receiving tetracycline? You answered this question Correctly 2. Take tetracycline on a full stomach. 1. , 3., 4., & 5. Correct: Avoid exposure to sunlight or artificial UV rays (sunlamps or tanning beds). Tetracycline can make your skin more sensitive to sunlight and sunburn may result. Use a sunscreen (minimum SPF 15) and wear protective clothing if you must be out in the sun. Take tetracycline on an empty stomach and do not take iron supplements, multivitamins, calcium supplements, antacids, or laxatives within 2 hours before or after taking tetracycline. These products can make this medicine less effective. Tetracycline can make birth control pills less effective. Use a second method of birth control while you are taking this medicine to keep from getting pregnant. Throw away any unused tetracycline when it expires or when it is no longer needed. Do not take this medicine after the expiration date on the label has passed. Expired tetracycline can cause a dangerous syndrome resulting in damage to the kidneys. 2. Incorrect: Take tetracycline on an empty stomach and do not take iron supplements, multivitamins, calcium supplements, antacids, or laxatives within 2 hours before or after taking tetracycline. These products can make this medicine less effective. Question: A 72 year old client admitted with a diagnosis of bleeding ulcers has been prescribed ranitidine 50 mg IVPB every 8 hours and omeprazole 10 mg po every morning. Based on this data what intervention should the nurse take first? You answered this question Correctly 2. Send the client for a CT scan of the head. 3. Provide oxygen at 2L/NC. 4. Notify the primary healthcare provider. 1. Correct: Ranitidine can cause confusion in the elderly as well as agitation. Stop the infusion, then notify the primary healthcare provider. 2. Incorrect: Confusion is a side effect of ranitidine, so the medication should be stopped and then the healthcare provider notified. 3. Incorrect: The oxygen saturation is not low at 95%. Providing oxygen will not correct the confusion. 4. Incorrect: The primary healthcare provider should be notified after stopping the ranitidine. Question: A nurse invites a friend home one evening. On arrival, the friend sees the nurse's large, white, long-haired cat sitting on the couch and begins to experience palpitations, trembling, nausea, shortness of breath, and a feeling of losing control. What should the nurse do first? You answered this question Correctly 1. Stay with the friend until the friend feels better. 2. Have the friend breathe into a paper bag. 4. Dim the lights in the room. 3. Correct: Remove the source of the panic attack first, then continue to assess the person for symptoms. Removing the cat "fixes" the problem. 1. Incorrect: This is a correct answer, but remove the cat first. 2. Incorrect: This will help hyperventilation if it occurs, but the stem of the question said the client was having "shortness of breath" related to seeing the cat. Remove the cat first as this will fix the problem and alleviate the symptoms. 4. Incorrect: This is a correct answer, but remove the cat first. Question: A client being treated for osteoporosis with alendronate reports experiencing slight heartburn after taking the medicine. What should the nurse suggest to reduce this side effect? You answered this question Correctly 1. Stop taking the medication and call the primary healthcare provider. 3. Take the medication before bedtime. 4. Take antacids when taking the medication. 2. Correct: Increased heartburn can be reduced or prevented by drinking plenty of water, sitting upright following the administration of the medication, and avoiding sucking on the tablet. 1. Incorrect: Slight heartburn does not warrant primary healthcare provider report. Extreme pain or difficulty swallowing should be reported, as should heartburn that increases despite suggestions listed above. 3. Incorrect: The client should take in the morning, thus preventing prolonged contact with the esophagus. 4. Incorrect: The absorption of the medication is decreased when it is taken with calcium, iron, and magnesium, or antacids containing calcium, aluminum, or magnesium. Thirty minutes should elapse before taking the antacid following administration of the alendronate. Question: The nurse determines that a client does not have an advance directive. The daughter is designated to make healthcare decisions in the event that the client becomes incapacitated or unable to make informed decisions. Which nursing actions are appropriate for this client? You answered this question Incorrectly 4. Avoid inquiring about a client's advance directive as this could cause the client anxiety and concern. 5. Ask the daughter if she agrees with her mother's decision. 1., 2. & 3. Correct: The nurse should document the client’s statement in the client’s own words. The nurse should provide the client with information on advance directives and assurance that there are hospital personnel to assist with completing the advance directive. 4. Incorrect: The nurse who avoids inquiry about a client’s advance directive is not serving the client’s best interests. The nurse should explain to the client that the law requires all clients be asked about the existence of an advance directive at the time of hospital admission. Preparing an advance directive ensures that the client’s wishes will be followed in the event that the client is unable to make healthcare decisions. 5. Incorrect: Providing information is the appropriate nursing action, not questioning the daughter. Question: Family members have been asking triage nurses if loved ones were admitted to the hospital during a national emergency situation with massive casualties. What response should be made by the nurses? You answered this question Correctly 1. Tell the family members that information about clients cannot be provided. 2. Ask for the victims’ permission before talking with the family members. 3. Instruct the family to wait for public announcements about victims. 4. Correct. The national emergency situation allows waivers for the Health Insurance Portability and Accountability Act (HIPAA) provisions. Due to the emergency situation, the nurse may inform the family members about the status of their loved ones. 1. Incorrect. During a national disaster declared by the President, information may be given to families without client consent. 2. Incorrect. There is no need to make the family more worried if information is known. Waivers for certain elements of HIPAA are allowed during the emergency period. 3. Incorrect. The nurse may legally give information to the family. The triage nurse may provide information concerning their family members Question: What precautions should be taken with computer monitors that display client health information to ensure client's confidentiality? You answered this question Incorrectly 1. Orient the screen facing the client rooms so that healthcare personnel can access the information easily. 3. Turn the computer monitors off when the computer is not in use. 4. The computer should be kept in a secured, locked area. 2. Correct: Computer monitors that display client health information should be positioned away from the view of any visitors or unauthorized persons. Even a well- guarded computer monitor, with an authorized employee sitting in front of it, could be a potential breach of confidentiality, depending on the angle of the monitor screen and who was attempting to view the information on it. The responsibility for keeping health information safe is on every member of the healthcare team. 1. Incorrect: No, this would be easily viewed by unauthorized individuals. 3. Incorrect: Not necessary to turn off if proper precautions are taken. 4. Incorrect: Not necessary to keep in secured area if proper precautions are taken. Question: A home health nurse has taught a client about home dressing changes using a clean technique. Which statement made by a client indicates to the nurse that the client understands this technique? You answered this question Correctly 1. “The wound should be cleaned using a washcloth, soap, and water.” 2. “Povidone-iodine should be applied to the wound with each dressing change.” 4. “I will use sterile gloves to clean my wound and change the dressings.” 3. Correct: Clean technique requires washing hands with soap and water prior to removing the dressing. 1. Incorrect: The wound should be cleaned with 4x4’s and sterile water. Soap can be very drying to the wound. A washcloth may not be clean as it has been sitting in a cabinet. 2. Incorrect: Povidone-iodine is harsh and damages healthy tissue, so should not be applied to the wound. 4. Incorrect: Sterile gloves are not needed when using clean technique. Clean gloves may be used. Question: The nurse is performing CPR on an adult client with facial and neck trauma. Following the administration of rescue breaths, where is the best location for the nurse to assess for a pulse in this client? You answered this question Incorrectly 1. Apical area 2. Carotid artery 4. Radial artery 3. Correct: Pulses that are best palpated are large and close to the trunk of the body. The femoral artery is large and at the trunk (proximal) of the body. 1. Incorrect: It would take too long to use a stethoscope and listen for an apical pulse on the client. A quicker area of checking the pulse would need to be used. 2. Incorrect: The client has bilateral neck trauma. The carotid artery would not be the best site to assess for a pulse. 4. Incorrect: The radial artery is not as large as the femoral artery and is distal to the femoral artery. Question: A client with heart failure and pulmonary edema is given furosemide intravenously. Which assessment indicates that the furosemide has achieved the desired effect? You answered this question Incorrectly 1. Weight has decreased 2 pounds. 2. Systolic blood pressure has decreased. 3. Urinary output has increased. 4. Correct: The goal for diuretic therapy in this client is to prevent/relieve fluid accumulation in the lungs. This answer addresses the most life-threatening sequelae with HF. The number one thing to worry about in clients with HF is pulmonary edema, because this is what can kill the client. 1. Incorrect: Weight loss is a good assessment of fluid loss or gain, especially acute weight changes. The stem of the question, however, asks which is the desired effect? The desired effect is to decrease fluid in the lungs and weight loss alone does not ensure that the lungs are clearing. 2. Incorrect: Lowered blood pressure is an expected finding, but treatment of pulmonary edema is the primary goal. 3. Incorrect: Increased urinary output is an expected finding, but treatment of pulmonary edema is the primary goal. Question: The nurse is planning health promotion strategies for an older client on a limited, fixed income who is trying to increase activity. The client has been cleared for moderate physical activity by the primary healthcare provider. Which strategies would be appropriate for this client? You answered this question Incorrectly 1. Suggest that the client join a local gym for access to equipment and support. 2. , 3., 4. & 5. Correct: The neighborhood buddy is accessible and can be a source of emotional support too, which increases the likelihood of continuing the plan. This activity is easily accessible and burns calories during the day or evening. Senior centers usually do not cost any money for the client, and other seniors may help motivate the client to increase activity level. The use of ordinary items does not further strain a fixed income. 1. Incorrect: Joining a gym will require monthly fees, thus impacting financial resources in a negative way. Additionally, transportation to and from the gym could impact finances in a negative way. Question: The primary healthcare provider prescribes glycopyrrolate 0.2 mg IM thirty minutes prior to electroconvulsive therapy (ECT). What should be the nurse's response when the client asks why this drug is being given? You answered this question Incorrectly 1. "The action of the medication is complex." 2. "This drug will prevent you from having a seizure." 3. "This medication will relax your muscles so that you do not break a bone." 4. Correct: Glycopyrrolate is an anticholinergic. Glycopyrrolate blocks the activity of acetylcholine which reduces secretions in the mouth, throat, airway, and stomach. It is used prior to procedures to decrease the risk of aspiration. 1. Incorrect: The client has a right to be told the reason the drug is given. This is a nontherapeutic communication response. The nurse should not refuse the client's desire to understand their medications. 2. Incorrect: Glycopyrrolate blocks the secretions in the mouth, throat, airway and stomach. The medication does not prevent the client having a seizure. The ECT will induce a seizure, which is the desire. 3. Incorrect: This is not the drug’s purpose so this would be incorrect information to give to the client. Question: A client who has been on bed rest for several days is ambulating for the first time with assistance. Prioritize the actions the nurse should take by placing them in order from first to last. You answered this question Correctly The Correct Order Your Selected Order In order to keep a client safe, the nurse should first assess the client's orientation to determine the client's ability to follow instructions. Second, to avoid orthostatic hypotension, the nurse should assist the client to sit on the side of the bed. Third, apply the gait belt to ensure safety while ambulating. Fourth, assist the client to stand for a few seconds. The fifth action is to ambulate in the room. Question: The nurse is caring for a client who has pneumonia and is dehydrated. The primary healthcare provider has prescribed IV fluids and IV antibiotics. Based on the primary healthcare provider’s prescription and oral intake, what would be the 24 hour intake for this client? Exhibit You answered this question CorrectlyEnter the answer for the question below. • Rationale • Strategies Answer: 3670_ mls Rationale: The intake would be calculated by adding the following: Azithromycin: 250 mls Ceftriaxone: 50 ml X 2 = 100 mls IV of D51/2 NS at 125 ml/hr: 125mls X 24 hours = 3000 mls Total oral intake is 270 +50 = 320 mls TOTAL INTAKE = 3670 mls Question: The nurse is caring for a client who is to receive an IV infusion of heparin. The client’s dose is based on a sliding scale prescription. What is the priority lab value to check before initiating the heparin infusion? You answered this question Correctly 1. PT and/or INR 3. Platelet count 4. WBC count 2. Correct: The activated partial thromboplastin time (aPTT) is a lab value used to assess pathways in the clotting cascade and is used to monitor Heparin therapy in clients. To maintain a therapeutic level of Heparin, the aPTT should be maintained at 1.5-2.5 times the normal range. 1. Incorrect: The PT and INR are lab values used to monitor warfarin therapy in clients 3. Incorrect: The platelet count measures an individual's total platelet count. Thrombocytopenia is a platelet count of < 100,000. Thrombocytopenia increases the risk of bleeding; however, since the aPTT directly measures heparin therapy, it is the priority value to monitor. 4. Incorrect: The White Blood Cells (WBC) are not involved in the body’s mechanism for clotting. Question: The nurse is caring for a client who has been diagnosed with dissociative identity disorder. What is the most appropriate short term goal? You answered this question Correctly 1. Recovery of memory deficits. 2. Demonstration of the ability to perceive stimuli correctly. 3. Elimination of causative phobia. 4. Correct: In order for therapy to be successful, the client must first acknowledge that there are multiple personalities within the client's personality. 1. Incorrect: This is related to a client with dissociative amnesia. 2. Incorrect: This is related to a client with disturbed sensory perception. 3. Incorrect: This outcome would not be related to this client. Question: A client with gestational diabetes delivers an infant with macrosomia. What is the most vital component of the infant's assessment for the nurse to perform? You answered this question Correctly 1. Evaluation of the infant for cephalhematoma. 2. Determining if the infant sustained a clavicle fracture. 3. Observing for arm movement to evaluate for facial palsy. 4. Correct: Infants of diabetic mothers are at risk for hypoglycemia following birth. Hypoglycemia can trigger seizures and cognitive deficits. 1. Incorrect: Assessing for cephalhematoma is important in macrosomia infants, but not as vital as ensuring stable glucose levels. 2. Incorrect: Assessing for clavicle fracture is important in macrosomia infants, but not as vital as ensuring stable glucose levels. 3. Incorrect: Assessing for facial palsy is important in macrosomia infants, but not as vital as ensuring stable glucose levels. Question: In what order, after initially washing hands, should the nurse change a dressing on an infected abdominal surgical wound that has a Penrose drain and a large amount of purulent drainage? Place in priority order from first to last. You answered this question Correctly The Correct Order Your Selected Order First, apply clean gloves. Second, remove soiled dressings. Third, discard soiled dressings and clean gloves in red bag. Fourth, don sterile gloves. Fifth, clean surgical wound with moistened sterile 4x4’s. Sixth, clean around Penrose drain using circular pattern inside to outside. Seventh, place dry, sterile 4x4’s over surgical wound and Penrose drain. Eighth, apply abdominal dressing pad. Question: When assessing a client, the nurse finds that in response to painful stimuli the upper extremities exhibit flexion of the arm, wrist, and fingers with adduction of the limb, while the lower extremity exhibits extension, internal rotation, and plantar flexion. How would the nurse accurately document this finding? You answered this question Correctly 1. Decerebrate posturing 3. Reflex posturing 4. Superficial posturing 2. Correct: This describes decorticate posturing because they are moving towards the core of the body. 1. Incorrect: Decerebrate posturing occurs when the client is stimulated, and teeth clench and the arms are stiffly extended, adducted, and hyperpronated. The legs are stiffly extended with plantar flexion of the feet. Abnormal extension occurs with lesions in the area of the brain stem. 3. Incorrect: There is no such condition as reflex posturing. This is a distractor and can be ruled out by focusing on the meaning of the terms and similarities in verbiage. 4. Incorrect: There is no such condition as superficial posturing. This is a distractor and can be ruled out by focusing on the meaning of the terms and similarities in verbiage. Question: The nurse is caring for a client admitted to the emergency department with a history of asthma. Which assessment findings would the nurse anticipate? You answered this question Correctly 3. 3 + pitting edema to ankles 4. Kussmaul respirations 1., 2., & 5. Correct: The client with asthma has a pattern of dyspnea (shortness of breath), chest tightness, coughing, wheezing, and increased respiratory rate. 3. Incorrect: There should be no dependent edema with asthma. 4. Incorrect: This respiration classification relates to metabolic acidosis and is seen in DKA. Question: A client received 2nd and 3rd degree burns on both arms and the anterior trunk when pouring gas on a burning trash pile. With the percentage of burns indicated, what should the nurse anticipate? You answered this question Correctly 1. Movement of fluid out of the cells into the vascular space. 3. Rapid fluid shift out of the vascular bed 48 hours after the burn. 4. Severe fluid volume excess in the first 24 hours after the burn. 2. Correct: Using the Rule of Nines, the client would have burned approximately 36% of the body. For burns greater than 20-25% of the total body surface area, the nurse should recognize that significant vascular damage occurs which causes increased permeability. The fluid leaks out of the vascular space and out into the tissues (3rd spacing). The client can go into a severe fluid volume deficit and shock. 1. Incorrect: The movement of fluid is out of the vascular space into the tissues, not out of the cells into the vascular space. 3. Incorrect: The majority of fluid shifts out of the vascular bed occur in the first 24 hours. The diuresis phase begins about 48 hours after the burn injury when fluid is returning to the vascular bed. 4. Incorrect: The client is at risk for fluid volume deficit (not fluid volume excess) in the first 24 hours as the fluid leaks out into the tissue. Question: What should the nurse document after a client has died? You answered this question Correctly 5. Primary healthcare provider’s prescriptions 1., 2., 3., 4., & 6. Correct: All of these should be documented. Also document consideration of and preparation for organ donation; family notified and decisions made; location of identification tags. 5. Incorrect: The primary healthcare provider’s prescriptions do not need to be documented after a client dies. Question: A nurse is teaching a group of small farm owners how to prevent pesticide exposure for field workers. What points should the nurse include? You answered this question Incorrectly 2. Wearing protective clothing while working in the field and at home. 5. Boiling all vegetables for a minimum of 5 minutes prior to eating. 1. , 3. & 4. Correct: The standard precautions of hand hygiene is recommended prior to eating. This will reduce the transmission of pathogens and possible pesticides. The pesticides may be present on their clothing, so the clothing and shoes should be removed. Fruits and vegetables should be washed to remove pathogens and pesticides. All points are important to include when teaching prevention of pesticide exposure. 2. Incorrect: Yes, wear protective clothing while working in the field, but it is not necessary to wear protective clothing at home. 5. Incorrect: No, it is not necessary to boil or cook vegetables to remove chemicals from fresh produce. Washing thoroughly with water is adequate. Question: A nurse is caring for a client who reports fatigue, weight loss, afternoon fevers, night sweats, cough, and hemoptysis. What interventions should the nurse initiate? You answered this question Incorrectly 2. Restrict fluid intake to 500 mL per day. 5. Do not allow visitors for 48 hours. 1. , 3. & 4. Correct: The nurse should suspect that the client is suffering from tuberculosis. Early pulmonary TB is asymptomatic. When the bacterial load increases, nonspecific symptoms of fatigue, weight loss, afternoon fevers, and night sweats may set in. As disease advances, cough, sputum production, and hemoptysis may appear. This client has the classic symptoms of TB and should be placed on airborne precautions. N95 respirator ensures that the nurse does not inhale the TB organism. Placing in a semi-Fowler's position reduces the work of breathing. 2. Incorrect: Unless contraindicated, 3-4 liters of fluid is needed per day to liquefy secretions. 5. Incorrect: Visitors are allowed if standard and airborne precautions are followed. Question: A hysterical college student arrives in the emergency department in bloody soiled clothing. The nursing assessment reveals facial bruising and multiple contusions consistent with the client's report of being raped. Which initial nursing intervention takes priority at this time? You answered this question Incorrectly 1. Notify police of the alleged rape. 2. Allow the client privacy to wash self. 3. Remove clothing and bag for evidence. 4. Correct: Although there are specific protocols that must be followed when dealing with rape clients, it is important to remember that this client had all control taken away during the attack. The emotional effects of rape are as traumatizing as the physical injuries. Allowing the client to first express emotions, such as fear or anxiety, returns a small amount of control to a situation in which the client has little or none. 1. Incorrect: The police will indeed be notified of the situation by the hospital personnel, but the nurse's initial priority is to focus on the client's needs, both physical and emotional. 2. Incorrect: The client will not be permitted to clean self until all evidence has been collected per protocol. However, initial contact between nurse and client should focus on more than just the physical aspects of the situation. 3. Incorrect: Collection of all evidence for the police is a crucial part of treating rape clients and will be completed according to protocols. But it is more important to remember that this client has already been violated during the attack. Removing clothing before addressing emotional needs may further exacerbate that sense of violation. Question: The nurse is caring for a client taking benazepril. Which symptoms would be important for the nurse to report to the primary healthcare provider? You answered this question Correctly 1. BP 150/108 decreases to 138/86 3. Serum sodium level of 139 mmol/L 2. , 4., & 5. Correct: Weight gain of 5 pounds in one week is a s/s of an adverse effect of ACE inhibitor use. Weight gain is a sign of fluid retention. Angioedema is an adverse effect of ACE inhibitors and can be life threatening. This should be reported immediately to the healthcare provider. The potassium level is too high. Hyperkalemia is an adverse effect of an ACE inhibitor and needs to be reported. 1. Incorrect: This is an expected response of an ACE inhibitor. ACE inhibitors block the normal effects of renin-angiotensin-aldosterone system, thereby decreasing the blood pressure. 3. Incorrect: The serum sodium level here is within normal limits. There is no need to report normal lab values. Question: What discharge instructions should the nurse provide to the client post abdominal hysterectomy? You answered this question Correctly 3. Swimming is allowed if staples were used to close the skin. 5. Apply moist heat to surgical site the first couple of days for pain relief. 1., 2., & 4. Correct: The client should get up and move to prevent complications such as deep vein thrombosis (DVT), pneumonia, constipation, etc. The healthcare provider should be notified if the surgical wound is bleeding, red and warm to touch or has a thick, yellow, or green drainage. Pressing a pillow over incision when coughing or sneezing will ease discomfort and protect the incision. 3. Incorrect: Do not go swimming or soak in a bathtub or hot tub until the primary healthcare provider says it is ok. You worry about infection. 5. Incorrect: In the first couple of days, an ice pack may help relieve some pain at the site of surgery. Remember NCLEX wants you to think safety first when it comes to the use of heat. Question: Which assignments would be most appropriate for the RN to delegate to an LPN/VN? You answered this question Correctly 1. Six year old with new onset diabetes. 3. Three month old admitted with severe dehydration. 2. ,4. & 5. Correct: The best assignments for the LPN/VN would be the child with pneumonia admitted two days ago and the child admitted for developmental studies. The twelve year old with post op wound infection taking oral antibiotics is also stable. 1. Incorrect: The diabetic requires much teaching and supervision. This is an unstable client that should not be assigned to an LPN/VN. 3. Incorrect: The child with dehydration will require close intravenous fluid (IVF) monitoring, assessment and evaluation of condition. This client is unstable and should not be assigned to an LPN/VN. Question: The nurse receives new healthcare provider prescriptions on a client diagnosed with Addison’s disease. Which prescriptions should the nurse recognize as being inappropriately written and requiring clarification from the prescriber? You answered this question Correctly 2. IV of normal saline at 125 mL/hr 3. MRI of pituitary gland 5. Dehydroepiandrosterone DHEA sulfate 5 mg by mouth every other day 1. & 4. Correct: Use “daily” or “every day”. QD is an unapproved abbreviation. T.I.W. stands for three times a week; however, it is an unapproved abbreviation. Use “three times a week”. 2. Incorrect: This is a correct action and is written properly. 3. Incorrect: The primary healthcare provider may suggest an MRI scan of the pituitary gland if testing indicates the client might have secondary adrenal insufficiency. This is an approved abbreviation. 5. Incorrect: This is written correctly and may be given to women to treat androgen deficiency. Question: The nurse is preparing to administer iron dextran IM. Which injection site would be best for administration? You answered this question Incorrectly 2. Vastus lateralis site 3. Rectus Femoris site 4. Deltoid site 1. Correct: This site would be used for Z track IM injections. Iron preparations are administered by the Z track technique, preferably in a large, deep muscle such as the ventrogluteal muscle. 2. Incorrect: The vastus lateralis site could be used in adults with enough muscle mass, but ventrogluteal is preferred site. 3. Incorrect: The rectus femoris site can be used in adults when other sites are no longer accessible. It is not the preferred site. 4. Incorrect: The deltoid is a small muscle that is not well developed in many clients. It is not a recommended site for Z track medication administration. Question: A client comes to the clinic reporting palpitations, as well as nausea and vomiting while taking metronidazole. The nurse notes that the client is flushed and has a heart rate of 118 bpm. Based on this information, what is the most important question for the nurse to ask the client? You answered this question Correctly 1. “Do you take metronidazole on an empty stomach?” 3. “How long have you had these symptoms?” 4. “What other medications are you currently taking?” 2. Correct: Flushing, nausea and vomiting, palpitations, tachycardia, psychosis are signs of disulfiram-type reaction seen when using products containing alcohol (cologne, after shave lotion, or path splashes) or ingesting alcohol products while taking metronidazole. 1. Incorrect: Although it is preferable to take metronidazole on an empty stomach, this is not the most important question to ask at this time. 3. Incorrect: How long the client has had these symptoms is not as important as whether the client is using any alcohol containing products. 4. Incorrect: Although the nurse needs to know what other medications the client is taking, it is not as important as knowing if the client is using any alcohol containing products. Question: Which task should the nurse perform first? You answered this question Correctly 2. Changing a colostomy bag that is leaking. 3. Performing an admission assessment on a client. 4. Administering pain medication to a postoperative client. 1. Correct: The tracheostomy tube must be suctioned to keep the client's airway open. Suctioning the tracheostomy should take priority. Remember, airway first. 2. Incorrect: The client may be uncomfortable from the colostomy bag leaking. This task can be delegated. The suctioning of the client does not have priority over airway. 3. Incorrect: Important, but not priority over airway. There is no indication from the question that the new client is in distress. The priority intervention is to maintain the airway. 4. Incorrect: Important, but it does not take priority over airway. Question: What should the nurse do first when caring for a client who is being admitted with a diagnosis of meningococcal meningitis? You answered this question Correctly 1. Assess neuro status. 2. Obtain health history. 4. Orient client to the room and procedures. 3. Correct: Although all the options are appropriate, the priority is to place the client on droplet precaution to prevent the spread of meningococcal meningitis. 1. Incorrect: Placing client in isolation should be done first (actually prior to arriving to room) since the client has a diagnosis of meningococcal meningitis. Assessment of the neuro status can be done next. 2. Incorrect: The health history can be obtained after placing the client on droplet precautions. 4. Incorrect: Orientation can occur after the initiation of droplet precautions. Question: The nurse is caring for a depressed client. The client has a flat affect, apathy, and slowed physical movement. The client has not bathed in several days and there is a malodorous odor noted. Which intervention would be most appropriate at this time? You answered this question Incorrectly 1. Explain the rules about daily showers. 2. Leave the client alone since there is slowed movement. 4. Ask when he or she would like to take a shower. 3. Correct: Depressed clients often have little energy to do or think. Give short, simple commands during this time. 1. Incorrect: Not very therapeutic. This is difficult to comprehend at this time. Give short, clear, simple commands. 2. Incorrect: Do not ignore the problem. You must do what is best for the client and this would not be the best decision. 4. Incorrect: The client will not want to do anything at this time. It will be put off and depressed client's often have difficulty making decisions. Question: A primary healthcare provider prescribed KCL 40 mEq in 100 mL NS to infuse over 30 minutes. What action should the nurse take? You answered this question Correctly 1. Administer the KCL through the lowest IV line port. 3. Mix KCL 40 mEq into the present infusing bag of NS when it reaches 100 mL. 4. Set the infusion pump to 100 mL / hour. 2. Correct: Potassium chloride should be diluted and administered to infuse no faster than 40 mEq per hour. So this prescription would infuse too quickly and may lead to life threatening arrhythmias. 1. Incorrect: This prescription should not be administered as written. The rate of infusion over 30 minutes is too fast. 3. Incorrect: This is dangerous. Nurses should not add KCL to an existing bag of infusing fluid. This prescription should not be administered as written. 4. Incorrect: The nurse should verify the rate prior to initiating the infusion. The rate of administration should be clarified with the primary healthcare provider. Question: A client is diagnosed with new onset grand mal seizures. Which nursing interventions should the nurse implement for this client? You answered this question Correctly 1. Have an unlicensed assisitve personnel stay with the client. 4. Keep a padded tongue blade at the bedside. 2. , 3., & 5. Correct: During a seizure these interventions will help to protect the client from injury. The client may strike the side rails. The bed should be placed in the low position in case the client falls out of the bed. The client would need assistance to the floor if a seizure starts while ambulating. 1. Incorrect: It is not necessary to have someone stay with this client at all times. Place a call light within reach, put the client close to the nurses’ station, and pad the side rails. Have the client call for assistance to bathroom. Maintain bed rest until seizures are controlled or ambulate the client with assistance to protect from injury. 4. Incorrect: Do not place a padded tongue blade in a client’s mouth during a seizure. The padded tongue blade could cause injury. Question: A client diagnosed with schizophrenia who is taking monthly haloperidol injections develops slurred speech, shuffling gait and drooling. Which prescribed PRN medication would the nurse administer? You answered this question Correctly 1. Lorazepam 2. Atropine 4. Chlorpromazine 3. Correct: These signs and symptoms are reflective of pseudoparkinsonism, a form of extrapyramidal side effects which are side effects of the haloperidol. An anticholinergic agent maybe used for treatment. This is an anticholinergic agent that may be used for extrapyramidal side effects. 1. Incorrect: This is a sedative/hypnotic or antianxiety agent. It is not used for treatment of extrapyramidal side effects. 2. Incorrect: This is an anticholinergic agent, but not one commonly used to treat pseudoparkinsonism, a form of extrapyramidal side effects. It is commonly used to treat arrhythmias and preoperatively to decrease secretions. 4. Incorrect: This is another antipsychotic medication. Question: A nurse is preparing a lecture about suicide. Which target audience would be most appropriate? You answered this question Incorrectly 2. Girl Scout leaders 3. Support group of divorced parents 4. Hispanic immigrant farm workers 1. Correct: Among those who commit suicide, young men between the ages of 15-24 are more likely to commit suicide than young girls and women. The best line of defense is to teach about the warning signs of suicide to high school teachers, students, and parents of teens and young adults, particularly male. 2. Incorrect: Although teenage girls may attempt suicide, they are less likely than males to use a lethal method. Additionally, participation in groups such as the scouts will provide support for girls. 3. Incorrect: Joining a support group will help eliminate stress of being a single parent. Young males are more likely to attempt suicide by lethal means. 4. Incorrect: Hispanics have a lower suicide rate than Caucasians. Question: The six bed Labor and Delivery area is full when the Emergency Department nurse calls for a bed for a woman reporting low back pain, pelvic pressure and increased vaginal discharge at 36 weeks gestation. Which would be the most appropriate action for the charge nurse? You answered this question Correctly 1. Transfer a G4P4 who delivered full-term twins one hour ago to the antepartum/postpartum floor. 3. Transfer an 8 hour postpartum G1P1 on Magnesium Sulfate for eclampsia from the LDR unit to the ante/postpartum unit. 4. Request that the new client be admitted to the antepartum/postpartum floor. 2. Correct: The client and newborn are not in any present distress. Also the delivery occurred 3 hours ago. This client would not be a risk and could be cared for on the antepartum/postpartum floor. 1. Incorrect: This client is at high risk for hemorrhage due to still being in the fourth stage of labor and over distention of her uterus with a term multiple gestation. This client needs to stay in current location for close monitoring. 3. Incorrect: Close monitoring and frequent vital signs are required since central nervous system alterations and respiratory depression are common side effects of Magnesium Sulfate. 4. Incorrect: This client has subtle signs and symptoms of preterm labor and needs close monitoring. Ante and postpartum unit is for stable clients. Question: The home health nurse is assessing the home environment for possible irritants that could increase/precipitate symptoms of respiratory problems. Which assessment questions would be important to determine level of risk? You answered this question Incorrectly 5. Is your water supply treated by a municipal agency? 1., 2., 3. & 4. Correct: Presence of wood smoke could increase respiratory problems. Poorly vented gas heaters could increase carbon monoxide in the environment. Use of solvents or other agents that produce irritating fumes could increase risk. The particles from the sanding could irritate the respiratory tract as well. Second-hand smoke is irritating to the respiratory tract. Aerosols could trigger respiratory problems. 5. Incorrect: Water safety would not necessarily increase respiratory risk. Question: Which electrolyte imbalance would be the nurse’s priority concern in the burn client? You answered this question Correctly 1. Hypernatremia 3. Hypoalbuminemia 4. Hypermagnesemia 2. Correct: Good job. When the cells lyse they release potassium, and then the serum potassium goes up. And if the kidneys stop, we are in real trouble. 1. Incorrect: Well this one does occur when the client becomes very dehydrated, but it’s not as dangerous as the potassium one. 3. Incorrect: Low albumin can cause problems keeping fluid in the vascular space, but albumin is not an electrolyte. 4. Incorrect: No, the magnesium doesn’t go up unless the kidneys shut down. Question: A case manager is evaluating a client diagnosed with hemiplegia due to a cerebral vascular accident who will need assistive devices upon discharge. Which devices should the case manager include for this client? You answered this question Correctly 6. Button closures on clothes 1., 2., 3., 4., & 5. Correct: The goal is to promote self-care by the client as much as possible. The case manager should evaluate the need for assistive devices to help with eating, bathing, dressing, and ambulating. The dinner plate food guard will prevent food from being pushed off the plate. The transfer belt will provide safety for the client to get into a chair or back in bed. A raised toilet seat makes it easier for the client to sit on the toilet without falling. The long-handled shoe horn allows the client to put on shoes without assistance. Wide grip utensils accommodate a weak grip. 6. Incorrect: It is hard for someone with hemiplegia to use buttons. Velcro fasteners are best. Question: A client with a history of congestive heart failure has an implantable cardioverter defibrillator (ICD) surgically implanted. What teaching points should the nurse provide the client prior to discharge? You answered this question Incorrectly 1. Avoid hot baths and showers. 2. Increase intake of leafy green vegetable products. 5. Driving is not recommended for 1 year after placement of an ICD. 3. , & 4. Correct: Magnets can deactivate the defibrillator. Other transmitter devices should also be avoided. Most arrhythmias need only one shock, but the healthcare provider should be notified when a shock is delivered so that monitoring can increase. 1. Incorrect: Hot baths or showers are not contraindicated with ICDs. 2. Incorrect: Increase of leafy green vegetable products would have no relation to the ICD but should be avoided if the client is on warfarin. 5. Incorrect: The client cannot drive for 6 months after implantation of an ICD and cannot drive for 6 months after any shock therapy from the ICD. Question: The oncoming nurse has just received report and is preparing to make initial rounds. Which postpartum client should the nurse see first? You answered this question Correctly 1. A primipara 6 hours postpartum saturating one peripad every two hours 3. A primigravida 12 hours postpartum with the uterine fundus at the umbilicus 4. A multigravida 72 hours postpartum with a brownish pink lochia discharge. 2. Correct: Intense perineal pain is a symptom of a perineal hematoma which is a medical emergency. 1. Incorrect: Expected findings for the postpartum period are described here. This is a normal peripad saturation and does not indicate a problem. 3. Incorrect: Expected findings for the postpartum period are described here. This is the proper position of the fundus 12 hours postpartum. 4. Incorrect: Expected findings for the postpartum period are described here also. A client postpartal 72 hours should have a brownish pink lochia discharge. Question: The nurse educator is teaching a group of nursing students about client advocacy. What should the educator tell the students are the consequences of failure to act as a client advocate? You answered this question Incorrectly 4. Suspension or loss of license to practice medicine. 1., 2., 3. & 5. Correct: The role of client advocate is a nurse’s responsibility. Failure to act as a client advocate could result in a range of complications for the client, including life-threatening or life-ending complications. Failure to act as client advocate exposes the nurse to liability, potential legal action against the nurse and/or healthcare facility, and potential suspension or loss of license to practice nursing. The client advocate protects client autonomy and right to make decisions. 4. Incorrect: The nurse does not have a license to practice medicine. The nurse cannot work outside of their scope of practice. This action may result in the possible susp Question: A nurse drops a bottle of IV fluid, which shatters on the floor in the hallway. What action should the nurse take? You answered this question Incorrectly 1. Notify housekeeping to clean up the spill. 3. Pick up the glass with gloved hands and dispose into a puncture resistant sharps container. 4. Use a wet mop to collect the glass and dispose of it in the garbage can. 2. Correct: The nurse must not be cut by the broken glass. Proper removal of glass includes using a dustpan and broom to collect the glass and disposing of it into a puncture resistant sharps container. 1. Incorrect: While waiting for housekeeping someone could fall or get cut. The nurse should initiate cleanup. 3. Incorrect: Hands are never used to pick up glass even if they are gloved because of the increased risk of getting cut. 4. Incorrect: A wet mop will not pick up the glass, and glass pieces will stick to a wet mop. Question: Which nursing task would be appropriate to delegate to an LPN/VN? You answered this question Correctly 2. Administer regular insulin IV to a client in diabetic ketoacidosis. 3. Monitor a client’s closed drainage unit (CDU) for tidaling. 4. Assess a client for tactile fremitus. 1. Correct: The LPN/VN has the knowledge and skill to obtain a wound culture. This is within the scope of practice for the LPN/VN. 2. Incorrect: The LPN/VN cannot administer IV medications to an unstable client. This client needs the RN for close monitoring. 3. Incorrect: The LPN/VN cannot monitor a chest tube and closed drainage unit. This is an RN task. 4. Incorrect: Assessment is the task of an RN. LPN/VN's can collect data, observe, and monitor the client. Question: A client has been prescribed vancomycin 1 gram IV every 12 hours for the treatment of methicillin-resistant staphylococcus aureus (MRSA). Which action by a new nurse when administering this medication would require intervention by the charge nurse? You answered this question Correctly 1. Dilutes medication in NS 100 mL. 2. Delivers medication via an IV pump. 4. Monitors IV site every 30 minutes during infusion. 3. Correct: This dose of medication should be delivered over at least 60 minutes to prevent hypotension and ototoxicity. 1. Incorrect: The minimum dilution for 1 gram is 100 mL, so this action does not need intervention. 2. Incorrect: This is a correct action by the new nurse. A pump is required to ensure that medication is not delivered too rapidly. 4. Incorrect: A peripheral IV site should be monitored for pain, redness or swelling prior to initiating the infusion and every 30 minutes until the completion of the infusion. Question: Which nursing action is likely to improve client satisfaction and demonstrate acts of beneficence? You answered this question Correctly 1. Allowing clients to make their own decisions about care 2. Answering all questions posed by client in an honest manner 3. Reporting faulty equipment to the proper departments 4. Correct: Sitting and listening demonstrates kindness and compassion that are consistent with the ethical term "beneficence." Beneficence is taking positive action to help others and a desire to do good which is the core principle of client advocacy. 1. Incorrect: Autonomy is the ethical principle illustrated here by supporting independent decision making with clients. 2. Incorrect: Fidelity is the ethical principle illustrated here and refers to the concept of keeping a commitment. It is based upon the virtue of caring. 3. Incorrect: Reporting faulty equipment is an act to promote nonmaleficence or to do no harm. This is the core of nursing ethics. Question: A nurse is educating the parents of a child with celiac disease. The nurse knows the teaching is successful if the parents choose which food for their child's dinner? You answered this question Correctly 1. Turkey and lettuce sandwich on rye bread 3. Chicken, vegetables and a whole wheat roll 4. Hotdog and baked beans 2. Correct: Steak and potatoes are gluten free. 1. Incorrect: Client’s with celiac disease should maintain a gluten free diet. Rye bread contains gluten. 3. Incorrect: Client’s with celiac disease should maintain a gluten free diet. Wheat contains gluten. 4. Incorrect: Client’s with celiac disease should maintain a gluten free diet. Processed meats such as hotdogs and most sausages contain gluten. Question: A 68 year old client was admitted two days ago to a long-term care facility. The client has chronic kidney disease, coronary artery disease and chronic obstructive pulmonary disease. Oxygen 2 L/per nasal cannula is being administered. Assistance is needed with activities of daily living. The primary healthcare provider visits today and writes new prescriptions. Who is the best person for the charge nurse to delegate carrying out these prescriptions? You answered this question Incorrectly 1. Unlicensed assistive personnel (UAP) 3. RN 4. Charge Nurse 2. Correct. All the nursing responsibilities associated with the healthcare provider’s prescriptions are within the scope of practice of the LPN/LVN. 1. Incorrect. Giving medications is out of the scope of practice of the Unlicensed assistive personnel (UAP). 3. Incorrect. All the nursing responsibilities associated with the healthcare provider’s prescriptions are within the scope of practice of the LPN/LVN. The RN would need to be assigned to more unstable clients than this one. 4. Incorrect. The charge nurse is responsible for assuring that all client care is provided during the shift, so carrying out these prescriptions is not the best use of time and resources available to the charge nurse. Question: Following surgery, a client refuses to ambulate as prescribed. What action should the nurse take? You answered this question Incorrectly 1. Notify the primary health care provider of client’s refusal to ambulate. 2. Offer the client pain medication. 4. Perform passive range of motion exercises. 3. Correct: The nurse should educate the client about complications that can be prevented with ambulation, such as constipation, pneumonia, or deep vein thrombosis (DVT). 1. Incorrect: The first action should be for the nurse to educate the client so an informed decision can be made. This would put the nurse in an advocacy role and would more likely persuade the client to comply with ambulation. 2. Incorrect: The first action should be for the nurse to educate the client so that he/she can make an informed decision. Offering pain medication would be appropriate if pain is impeding the client's ability to move; however, pain medications may make the client at risk for falls so safety precautions would be priority. 4. Incorrect: The first action should be for the nurse to educate the client so that an informed decision can be made. Passive range of motion would not be the best option at this time. Question: The nurse recognizes that treatment has been successful in resolving fluid volume excess based on which assessment findings? You answered this question Correctly 1. Continued lethargy 2. Heart rate 112/min 5. Increased thirst 3. & 4. Correct: Urinary output should increase with decreasing shortness of breath as hydration is corrected, and BP should be normal. 1. Incorrect: Level of consciousness (LOC) should improve with perfusion to the brain. 2. Incorrect: Heart rate should decrease if hydration is corrected. 5. Incorrect: Thirst level should be decreased if hydration is corrected. Question: A client has been on the mental health unit for three days and is requesting to leave against medical advice (AMA). It has been determined that the client is not suicidal. What should the nurse do? You answered this question Correctly 2. Make arrangements for a commitment hearing, as soon as possible. 3. Tell the client the primary healthcare provider must discharge the client prior to leaving. 4. Call the primary healthcare provider and request a discharge order. 1. Correct: Protocols on the unit must be followed when someone is requesting to leave AMA. The first step is to call the primary healthcare provider about the client's desire to leave AMA. 2. Incorrect: The client is not suicidal; however, the primary healthcare provider needs input into their decision to leave AMA. It is not appropriate to prepare for a commitment hearing. 3. Incorrect: If the client is not a threat or potential threat to self or others, the client may leave. The nurse may discuss the decision to leave; however, this statement is not accurate. 4. Incorrect: The nurse should call the primary healthcare provider and discuss the situation. The primary healthcare provider should have input into this decision. Question: A client is reporting pain rated an 8 out of 10 on the numeric pain scale. The nurse administers an oral pain medication to the client and starts a CD of the client’s favorite relaxing music. Fifteen minutes later, the client rates the pain as 2 out of 10 on the numeric pain scale. What type of nonpharmacologic pain relief intervention has the nurse used? You answered this question Correctly 2. Biofeedback 3. Progressive relaxation 4. Cutaneous stimulation 1. Correct: The nurse uses distraction in the form of music while the oral analgesic takes effect. 2. Incorrect: Biofeedback is a behavioral therapy that trains individuals to take control of the physiological responses to stressors. 3. Incorrect: Progressive relaxation uses a combination of breathing exercises and muscle group contractions and relaxation. 4. Incorrect: Cutaneous stimulation uses stimulation of the skin through heat, cold, or even electrical nerve stimulation to decrease or eliminate pain. Question: The nurse is caring for a Native American client who has returned to the surgical floor following abdominal surgery. The nurse is concerned about the level of discomfort that the client is experiencing. Which comment is the nurse likely to hear upon assessment? You answered this question Correctly 1. "The pain is getting worse. I can’t stand it." 2. "I need something for pain as soon as possible." 3. "I hope that the pain will go away soon." 4. Correct: The Native American client is likely to be quiet and less expressive of pain. Native Americans tend to tolerate high levels of pain. 1. Incorrect: This comment is likely to come from the dominant American culture where pain is considered something to be treated. 2. Incorrect: Native Americans tend to tolerate high levels of pain. Abdominal surgery usually results in sensations of pain for most people. 3. Incorrect: The Native American client is likely to be very quiet about the pain being experienced. Question: Post cataract removal a client reports nausea and severe pain in the operative eye. Which nursing intervention takes priority? You answered this question Correctly 1. Administer morphine and ondansetron. 2. Reposition client to non-operative side. 3. Massage the canthus to unblock the lacrimal duct. 4. Correct: Severe pain with nausea indicates an increase in intraocular pressure and needs to be reported at once. Eye damage can result if not resolved quickly. The primary healthcare provider may prescribe medications or take the client back to surgery. 1. Incorrect: This is not the priority as severe pain and nausea indicates an increase in intraocular pressure and needs to be reported at once. 2. Incorrect: Repositioning will not fix the problem. Severe pain with nausea indicates an increase in intraocular pressure and needs to be reported at once. 3. Incorrect: The problem is an increase in intraocular pressure which needs to be reported to the primary healthcare provider. Question: Which nursing intervention should receive priority after a client has returned from having had eye surgery? You answered this question Correctly 1. Administer pain medication around the clock. 3. Apply warm compresses. 4. Instruct on importance of turning, coughing, and deep breathing. 2. Correct: Maintaining head of bed in an elevated position will help to decrease intraocular pressure. Do not lie the client supine as this will increase intraocular pressure. If the intraocular pressure increases too much, damage to the eye structures, including the retina and optic nerve, with resulting loss of vision, may result. 1. Incorrect: Although pain management is important, it is not the priority here. The priority intervention of maintaining the bed at 35° is to reduce the risk of increased intraocular pressure. Unless the pain becomes out of proportion or suddenly worsens, it is an expected finding and would not be cause for alarm or require "priority" attention. 3. Incorrect: Warm and hot compresses could possibly increase intraocular pressure and cause damage to the eye structures. 4. Incorrect: Coughing will increase intraocular pressure and could result in damage to the surgical site and/or the structure within the eye. Loss of vision could result if pressure becomes too great. Coughing is a type of valsalva movement which results in an increase in the intraocular pressure Question: A client is scheduled for plateletpheresis. When taking the client’s history, which information is most significant? You answered this question Correctly 1. Allergies to shellfish 3. Time of last oral intake 4. Blood type 2. Correct: Platelet donors can have plateletpheresis as often as every 14 days. 1. Incorrect: Allergies to shellfish have nothing to do with withdrawing platelets from the client. 3. Incorrect: Time of last oral intake has no bearing on whether or not a client can donate platelets. 4. Incorrect: Blood type has no bearing on whether or not a client can donate platelets. Question: The charge nurse on the pediatric unit is reviewing the protocol for blood administration with a staff nurse. Which actions by the staff nurse indicate understanding of blood administration? You answered this question Correctly 1. The blood infusion time was within 6 hours. 5. One form of client identification were obtained prior to infusion. 2. , 3., & 4. Correct: Filters are used when infusing blood. Two nurses must check the blood product label and blood group. Vital signs are checked frequently during a blood transfusion. For example: A baseline set of vital signs are taken, then again 5 minutes after the initiation of the transfusion, then 15 minutes after transfusion started and every 15 minutes for one hour, then every 30 minutes for one hour, then hourly until infusion is complete. 1. Incorrect: Blood should hang for no longer than 4 hours because it increases the chances of a reaction. 5. Incorrect: At least two methods of proper identification should be obtained, such as asking client his/her name and checking ID band. Question: The home health nurse is caring for a client with a neurological urinary tract dysfunction. What information should be included when teaching the client how to perform intermittent self catheterization? You answered this question Correctly 1. Performed in an emergency department (ED). 2. Prevents urinary catheter infections. 4. Requires using sterile gloves. 3. Correct: Home intermittent catheterization is a clean, not sterile technique when performed in the home environment. Home intermittent catheterization is preferred over continuous use of an indwelling catheter, as an indwelling catheter increases client risk of urinary tract infection (UTI). 1. Incorrect: The client can be taught to do self-catheterization at home. The client does not need to go to the emergency department (ED) to perform the self- catheterization procedure. 2. Incorrect: Performing intermittent self-catheterization at home is recommended for urinary retention. It does not prevent urinary tract infections. 4. Incorrect: Intermittent self-catheterization is a clean procedure, not sterile technique. Question: The nurse is searching for information about the nursing care of a client receiving an experimental drug for the treatment of obesity. Which database is most likely to address this issue? You answered this question Incorrectly 2. Cochrane Library 3. Health and Wellness Resource Center 4. MEDLINE 1. Correct: The Cumulative Index for Nursing and Allied Health Literature (CINAHL) is a source for reviewing nursing and allied health information. It is also located in other healthcare data bases. 2. Incorrect: Cochrane Library includes evidence based medicine databases. 3. Incorrect: The Health and Wellness Resource Center provides access to a variety of journal articles, magazines, and pamphlets. 4. Incorrect: MEDLINE is one of the major sources for biomedical information. Question: A client diagnosed with arachnophobia is prescribed alprazolam 0.5 mg orally three times daily. The nurse knows that teaching about this medication is successful when the client makes what statement? You answered this question Correctly 1. Alprazolam will take up to two weeks to start working. 2. The drug does not cause drowsiness, so my daily activities will not suffer. 3. This medication cannot be taken with food. 4. Correct: Suddenly stopping could produce serious withdrawal symptoms, such as depression, insomnia, anxiety, abdominal and muscle cramps, tremors, vomiting, sweating, convulsions, and delirium. 1. Incorrect: Alprazolam works relatively quickly. 2. Incorrect: Drowsiness, confusion, and lethargy are the most common side effects. The client should not drive or operate dangerous machinery while taking the medication. 3. Incorrect: If the client experiences nausea and vomiting, take with food or milk. Question: The primary healthcare provider prescribed tolbutamide 250 mg orally twice a day. The pharmacy dispensed tolbutamide 0.5 g scored tablets. How many tablets will the nurse administer? Round your answer using one decimal point. You answered this question CorrectlyEnter the answer for the question below. Step 1: 1000 mg : 1 g = x mg: 0.5 g x = 500 mg Step 2: 500 mg: 1 tab = 250 mg : x tab 500 x = 250 X = 0.5 Question: Which signs and symptoms, if noted by the nurse, would indicate that the client with hyperthyroidism is experiencing thyroid crisis? You answered this question Incorrectly 2. Bradycardia 1. , 3., 4., & 5. Correct: These are symptoms of thyroid crisis and should be reported immediately. 2. Incorrect: Tachycardia would occur, not bradycardia. Bradycardia is a symptom of hypothyroidism. Question: A clinic nurse is educating a client diagnosed with Bell’s Palsy. What is the most important educational point the nurse must emphasize to the client? You answered this question Correctly 1. Physical therapy will be needed to maintain muscle tone of the face. 2. Massaging the face several times daily using a gentle upward motion. 4. Non-steroidal anti-inflammatory medications are used to alleviate painful muscles. 3. Correct: Even though all are educational points that need to be provided to the client, this is the most important educational point to make. Keratitis, or the inflammation of the cornea, is one of the most dangerous complications for a client with Bell’s palsy. As a precautionary measure, the nurse must ensure that the cornea is protected even if the eyelids will not close. 1. Incorrect: Physical therapy will be needed, however care to prevent eye injury takes priority. 2. Incorrect: This can be done once the client’s facial sensitivity to touch decreases and the client can tolerate touching the face. Preventing eye injury takes priority. 4. Incorrect: This is true, but again, protecting the eye is the most important point to convey to the client. Question: A long-term care nurse is planning care for a newly admitted client diagnosed with Alzheimer’s disease. What should the nurse include in the plan of care? You answered this question Correctly 2. Perform activities of daily living for the client. 1. , 3., 4., & 5. Correct: All of these should be included in this client’s plan of care. Assess the client’s ability to perform activities of daily living and allow client to perform alone if capable. Maintain stimuli such as a clock, newspaper, calendar, and/or weather status. Encourage family to visit to maintain socialization. Plan for staff to spend some time talking and listening to the client. 2. Incorrect: Teach staff to facilitate client’s independence in all activities for as long as they are able. Encouraging self-care and allowing the client to be involved in activity of daily living (ADL) is a basic right and core principle of planning client-centered care. Question: The client has just returned from electroconvulsive therapy (ECT) and is very drowsy. What is the position of choice for the nurse to place the client in until full consciousness is regained? You answered this question Correctly 1. Supine 2. Fowler's 4. High Fowler's 3. Correct: When someone is very sedated and not fully conscious, we want them on their side so the airway remains open and the secretions can drain. 1. Incorrect: No, the jaw will fall back, the tongue will block the airway, and the client will have airway obstruction, either partial or maybe even life-threatening. 2. Incorrect: No, if you sit a client up who is not fully conscious, the client's head tips forward and blocks the airway. 4. Incorrect: Again, head may fall forward and block airway. Question: What symptoms would the nurse anticipate in a client being admitted to the hospital with a calcium level of 3.2 mg/dL (0.80 mmol/L)? You answered this question Correctly 1. Slowed deep tendon reflexes. 3. Hypoactive bowel sounds. 2. , 4., 5., & 6. Correct: Normal serum calcium is 8.7 – 10.3 mg/dL (2.18 – 2.58 mmol/L). The client with a calcium level of 3.2 mg/dL (0.80 mmol/L) is hypocalcemic. With hypocalcemia, the muscle tone is rigid and tight. Therefore, the client may report muscle cramping. A hallmark sign of hypocalcemia is a positive Chvostek's sign, which is a twitching of facial muscles following tapping in the area of the cheekbone, indicative of hyperirritability. The client may be at risk of having seizures due to the neuromuscular irritability. Prolonged contraction of the respiratory and laryngeal muscles causes laryngospasm and stridor and may result in cyanosis. 1. Incorrect: With hypocalcemia, the deep tendon reflexes are increased, not decreased. 3. Incorrect: The client with hypocalcemia tends to have increased gastric motility, resulting in hyperactive (not hypoactive) bowel sounds. In addition, abdominal cramping and diarrhea are common with hypocalcemia. Question: Which action by a nurse indicates to the charge nurse that the sterile field has been contaminated? You answered this question Correctly 1. The sterile field is above the level of the waist. 3. Remains facing the sterile field throughout procedure. 4. Inspects sterile wrapped instruments for tears. 2. Correct: No sterile object should be within the one inch border of the sterile field as the object is no longer considered sterile. 1. Incorrect: This is a correct action. Bacteria tend to settle below the level of the waist, so there is less contamination when the field is above the waist and away from the nurse. 3. Incorrect: This is a correct action. The nurse should never turn their back to the sterile field. The fronts of sterile gowns are considered sterile from the chest to the level of the sterile field. 4. Incorrect: This is a correct action. The sterile wrapped instruments and trays should be purposely inspected for small tears that would compromise sterility before opening and placing the instruments on the field. Question: A school nurse educates a group of teachers how to extinguish a fire involving a child whose clothes are on fire. Which statement by the teachers would indicate to the school nurse that the teachers understand what should be done first? You answered this question Correctly 1. “Someone should be assigned to call 911.” 3. “A blanket should be thrown over the child’s head and body.” 4. “Use a fire extinguisher to put out the flames.” 2. Correct: The flames should be extinguished first. The best way to accomplish this it to lay the child flat and roll in a blanket. This is referred to as the drop and roll method, when a blanket is available. 1. Incorrect: 911 should be called but the most important thing to do is to extinguish the flames first. 3. Incorrect: Throwing a blanket over the child’s head can trap smoke. This may lead to smoke inhalation and does not extinguish the fire. 4. Incorrect: The fastest and most effective way to extinguish the flames is by using the drop and roll method. Question: An emergency department (ED) nurse working triage has assessed four clients. Which client should receive the highest priority? You answered this question Incorrectly 1. Alert client who fell on the side walk. Skin warm and dry to the touch, with a three inch laceration on the right knee continuously oozing dark red liquid. 3. A client who "passed out" but regained consciousness when feet were elevated. Awake and confused, with warm and dry skin. 4. An alert, responsive client who reports severe abdominal and shoulder pain that began two hours after eating at a local fast food restaurant. Skin is warm and dry. 2. Correct: This client is responding to verbal stimuli by moaning and has an open airway; but any client with an altered level of consciousness is at risk for airway obstruction. The skin assessment indicates a circulation problem. 1. Incorrect: This client demonstrates no signs of life threatening problems. The client is stable. 3. Incorrect: This client has an open airway and the skin findings do not suggest a circulation problem. This client is confused but alert, so lower priority. 4. Incorrect: The client is alert and talking so the airway is open. The client’s skin findings do not suggest a circulation problem. This client has no immediate life- threatening problems. Question: A new admit arrives to the nursing unit with one thousand dollars in cash. What would be the best action by the nurse to safeguard the client’s money? You answered this question Correctly 1. Insist the money go home with the client’s visitor. 2. Place the money in the client’s bedside table drawer. 4. Lock money up in narcotic cabinet with client’s identity and room number. 3. Correct: The best action by the nurse would be to itemize the valuables, place in an envelope, and put in the hospital safe. 1. Incorrect: This is not the best option. The visitor may not be the best person to take the money. The client also has the right to refuse. 2. Incorrect: This is not a safe option. Anyone could retrieve the money. 4. Incorrect: This is not a safe option. Anyone with access could retrieve the money. Question: The nurse is assigned to care for a client who has developed intestinal obstruction and has had an NG tube inserted to low suction. Blood gases are pH 7.54, pCO2 52, HCO3 35. Assessment of the client by the nurse reveals that the client is weak, shaky, and reporting tingling of the fingers. The nurse determines that this client is in which acid/base imbalance? You answered this question Correctly 1. Respiratory acidosis 2. Respiratory alkalosis 3. Metabolic acidosis 4. Correct: Metabolic alkalosis occurs from gastric losses via vomiting, NG tubes to suction, or lavage, and potent diuretics. Signs and symptoms include n/v, sensorium changes, tremors, convulsions. pH > 7.45, pCO2 > 45, HCO3 > 27. 1. Incorrect: Not a respiratory related acid/base imbalance. 2. Incorrect: Not a respiratory related acid/base imbalance. 3. Incorrect: Not acidosis. There is loss of gastric acid. Question: A primary healthcare provider informs the nurse to prepare for an amniotomy on a client who’s labor has not progressed. What should the nurse assess for prior to the primary healthcare provider performing this procedure? You answered this question Correctly 1. Fetal attitude 3. Fetal lie 4. Fetal position 2. Correct: Fetal engagement is important prior to rupturing the membranes so that the umbilical cord cannot prolapse. Fetal engagement is when the fetus is at station 0 (level of mom’s ischial spines). 1. Incorrect: Fetal attitude is where the extremities and chin of the fetus are in relation to the fetal body. 3. Incorrect: Fetal lie refers to the maternal spine in relation to the fetal spine. 4. Incorrect: Fetal position tells us the presenting part of the fetus to mom’s pelvis. Question: Which task would be appropriate for the nurse to delegate to an unlicensed assistive personnel (UAP)? You answered this question Correctly 1. Check client for signs of skin breakdown. 3. Administer 8 ounces of polyethylene glycol electrolyte solution every 10 minutes. 5. Determine what activities the client can do independently. 2. & 4. Correct. These tasks are within the scope of practice for the UAP. 1. Incorrect. The UAP cannot assess the client for signs of skin breakdown. 3. Incorrect. The UAP cannot administer medication. 5. Incorrect. The UAP cannot assess which activities that the client can perform. Question: The nurse is caring for a client following spinal surgery. The client is placed on methylprednisolone. What additional drug therapy would the nurse expect to be prescribed with methylprednisolone? You answered this question Incorrectly 2. Phenytoin 3. Imipramine HCI 4. Aminocaproic acid 1. Correct: A potential side effect of methylprednisolone is a peptic ulcer. The primary healthcare provider will prescribe a proton pump inhibitor or H2 blocker to prevent this side effect. 2. Incorrect: Phenytoin is an anticonvulsant. Seizures are not a side effect of methylprednisolone. 3. Incorrect: Imipramine HCI is an antidepressant which is not routinely given with methylprednisolone (Although mood changes can occur with steroid administration, anti-depressants are not routinely given). 4. Incorrect: Aminocaproic acid is given when clients are bleeding. Bleeding is not a side effect of methylprednisolone. Question: A client is hospitalized because of severe malnutrition related to anorexia nervosa. What is the most important goal for this client? You answered this question Correctly 1. Verbalize understanding that eating behaviors are maladaptive. 2. Verbalize the importance of adequate nutrition. 4. Acknowledge misperception of body image as fat. 3. Correct: Until appropriate weight is gained, the client continues to be at risk for major health complications including hypotension, cardiac arrhythmias, poor muscle tone, increased risk for infection, abnormal liver function, and damaged kidneys. 1. , 2. & 4. Incorrect: These are all appropriate goals; however, the most important one is that the client gain adequate weight. Question: The nurse is caring for a client with decreased cardiac output secondary to heart failure with fluid volume overload. Which signs/symptoms are an indication to the nurse that treatment goals have not been met? You answered this question Correctly 1. Diuresis 4. Warm, dry skin 6. Alert and oriented 2. , 3. & 5. Correct: When the cardiac output decreases, renal perfusion decreases, which leads to decreased urine output and fluid retention. This leads to difficulty breathing. Tachycardiac and rhythm irregularity are signs of fluid volume excess (FVE) and decreased output. Persistent cough, wheezing, and pink blood tinged sputum are all signs that the client is still sick. 1. Incorrect: No, that is what we want, but there is not enough blood to the kidneys, and the renin angiotensin (aldosterone) mechanism has activated. 4. Incorrect: Indication of improved cardiac output. 6. Incorrect: Indication of improved cardiac output. Question: The nurse is caring for a client with possible hepatic failure. The nurse asks the client to sign a permit for a procedure. The nurse recalls the client's admission signature as legible, but, now observes a jerky, illegible signature. What should the nurse suspect is the cause of this handwriting change? You answered this question Correctly 1. Fetor 2. Ataxia 3. Apraxia 4. Correct: Yes, the liver flap, which is an abnormal muscle tremor, is usually found in clients with diseases of the liver. This is an indication that the hepatic failure is worsening and needs to be documented. 1. Incorrect: This is an offensive odor of the client's breath often associated with liver failure. 2. Incorrect: This is uncoordinated movement that is associated with many different neuromuscular disorders. 3. Incorrect: This is a term to describe not using items for their intended purpose and is associated with neurological disorders and damage to the brain. Question: Which statement by a student nurse indicates to the nurse educator that teaching regarding witnessing consent signatures has been successful? You answered this question Correctly 1. “Two people must witness a consent signature.” 2. “A RN must witness a consent signature.” 4. “A witness must be over the age of 21.” 3. Correct: Signing as a witness implies that the witness has observed the client personally signing the consent form with no coercion. 1. Incorrect: Only one signature is required as a witness. 2. Incorrect: The witness does not have to be an RN. 4. Incorrect: A witness is required to be over the age of 18. Question: The charge nurse was notified that a client with 2nd degree burns is being admitted to the floor. Which nurse should be assigned this client? You answered this question Incorrectly 1. A nurse caring for clients with spina bifida and acute gastroenteritis. 2. The new nurse, out of orientation for 2 months, caring for clients diagnosed with tonsillitis and anorexia nervosa. 3. The pregnant nurse caring for clients with cystic fibrosis and staph infection. 4. Correct: It would be best to assign the client to this nurse because the clients this nurse is caring for do not have anything contagious, which will decrease the risk of the burn client becoming infected. 1. Incorrect: This nurse is caring for a client with infection: acute gastroenteritis. 2. Incorrect: This nurse is caring for a client with infection: tonsillitis. 3. Incorrect: This nurse caring for a client with infection: staph infection. Question: A nurse is caring for a client in an outpatient clinic. The client lost her husband of 51 years three months ago. Which findings support that the client is experiencing normal grief reactions rather than clinical depression? You answered this question Correctly 1. The client is experiencing anhedonia. 4. The client has a persistent state of dysphoria. 2. , 3. & 5. Correct: A client going through a normal grieving process will experience a mixture of good and bad days. The client experiences moments of pleasure and cries less. 1. Incorrect: Anhedonia is the inability to experience pleasure seen in clinical depression. This would not be a positive sign of normal grieving in a client. 4. Incorrect: Dysphoria is a mood of general dissatisfaction, restlessness, depression, and anxiety. This is often seen in clinical depression. Question: When caring for a client with hepatitis A, the nurse should take what special precaution? You answered this question Correctly 1. Wear gloves when handling blood and body fluids. 2. Wear a mask and gown before entering the room. 4. Use caution when bringing food to the client. 3. Correct: Hepatitis A is transmitted by the fecal/oral route. 1. Incorrect: Hepatitis B is transmitted by contact with blood and body fluids. 2. Incorrect: Masks are not necessary when a client is placed on contact isolation. Hepatitis A is NOT transmitted by airborne or droplet transmission. 4. Incorrect: Use caution in bringing fresh produce to clients placed on reverse isolation. Question: Which assessment findings would be of concern to the nurse who is caring for a client who has an arterial line to the radial artery? You answered this question Incorrectly 3. Warm skin to right and left hand. 1., 2., 4., & 5. Correct: These are all signs of poor circulation to the right hand. The arterial line could be occluding circulation and needs to be removed. 3. Incorrect: This is a good sign. We would worry with cool skin/extremity. Question: A client diagnosed with mania and hypertension is hospitalized due to confusion and polyuria. Based on current data, what interventions should the nurse implement? You answered this question Incorrectly 4. Administer sodium polystyrene for hyperkalemia. 1., 2., 3., & 5. Correct: Symptoms of lithium toxicity begin to appear at blood levels greater than 1.5 mEq/L. Additionally, concurrent administration of lithium and diuretics such as furosemide increase the chance of toxicity. At serum levels of 1.5-2.0 mEq/L: blurred vision, ataxia, tinnitus, persistent nausea and vomiting, severe diarrhea. At serum levels of 2.0-3.5: excessive output of dilute urine, increasing tremors, muscular irritability, psychomotor retardation, mental confusion, giddiness. At serum levels above 3.5: impaired consciousness, nystagmus, seizures, coma, oliguria/anuria, arrhythmias, myocardial infarction, cardiovascular collapse. Arrhythmias and seizures can occur with toxicity. So the lithium dose should be held, and the healthcare provider notified. The client is at risk for arrhythmias, so connect to a heart monitor. The client is also at risk for seizures, so pad the side rails. 4. Incorrect: The potassium level is normal, so there is no need to treat hyperkalemia. Question: A nurse attaches a client to continuous cardiac monitoring due to a potassium level of 2.8 mEq (2.8 mmol). The nurse should monitor for which dysrhythmia? You answered this question Incorrectly 1. Third degree heart block 2. Atrial fibrillation 3. Premature atrial contractions 4. Correct: Hypokalemia is reflected by the EKG. The earliest EKG change is often premature ventricular contractions (PVCs) which can deteriorate into ventricular tachycardia or fibrillation (VT/VF) without appropriate potassium replacement. 1. Incorrect: Atrial-ventricular blocks are not often seen initially with hypokalemia. 2. Incorrect: Atrial fibrillation is not often seen with hypokalemia. 3. Incorrect: PACs are not often seen initially with hypokalemia. Question: The nurse is teaching a diabetic client who has been prescribed Lispro insulin about avoiding hypoglycemia. What administration teaching is priority? You answered this question Incorrectly 1. Take insulin 30 minutes before bedtime 2. Take insulin twice daily in AM and PM 3. Take insulin one hour before meals 4. Correct: Lispro is a rapid-acting insulin that should only be taken with food or within 15 minutes of a meal. 1. Incorrect: Lispro is a rapid-acting insulin and should not be taken without food. 2. Incorrect: Lispro is a rapid-acting insulin and should not be taken without food. 3. Incorrect: Lispro is a rapid-acting insulin and should not be taken without food. Giving an hour before eating is too early and would put the client at risk for hypoglycemia. Question: The client in the manic phase of bipolar disorder begins climbing onto a table in the dayroom and shouts, "I can fly! I can fly! Watch me fly!" What should be the initial intervention by the nurse? You answered this question Correctly 1. Leave the client alone and remove clients from the dayroom. 3. Restrain the client, and notify the primary healthcare provider. 4. Tell the client that there is no way that a person can fly. 2. Correct: The first priority is the safety of the client. If the client jumps off the table, this action may cause an injury to the client. The nurse may need extra help in case the client becomes violent. 1. Incorrect: The client may cause self injury by jumping from a table. The other clients are not in danger. 3. Incorrect: The primary healthcare provider does not need to be notified at this time. Restraints should be used as a last resort. 4. Incorrect: Presenting a client with reality is a therapeutic communication technique. The safety of the client at this time is the priority intervention. Question: The nurse is caring for a client who has taken an acetaminophen overdose. Which symptom is the client most likely to exhibit? You answered this question Correctly 1. Expectorating pink frothy sputum 2. Sudden onset of mid-sternal chest pain 4. Diaphoresis and fever 3. Correct: This is a sign of liver damage, which is caused by an overdose of acetaminophen. 1. Incorrect: This is a symptom of pulmonary edema, not liver damage. 2. Incorrect: This is a symptom of myocardial ischemia, not liver damage. 4. Incorrect: Acetaminophen would decrease fever, and fever could cause diaphoresis so neither of these are expected with acetaminophen overdose. Question: The nurse notes that a client has impaired swallowing as a result of a cerebrovascular accident (CVA). Which interventions are appropriate for the nurse to include in the plan of care? You answered this question Correctly 2. Assist the client to hyperextend the head when preparing to swallow. 5. Start the client on a thin liquid diet. 1. , 3. & 4. Correct: This is the optimal position for chewing and swallowing without aspirating. Sitting up 30 minutes after completing a meal will prevent regurgitation of food. In case of choking, family members should know how to perform emergency measures such as the Heimlich maneuver. 2. Incorrect: The client should position the head in forward flexion in preparation for swallowing, called the “chin tuck”. Hyperextension would cause aspiration. 5. Incorrect: The client should be started on thick liquid or pureed diet. Thickened or pureed foods are easier to swallow than thin liquids and prevent aspiration. Question: The nurse is evaluating dietary education provided to a client diagnosed with cholecystitis. The nurse determines that further teaching is necessary when the client chooses which meal to consume? You answered this question Correctly 1. Cup of oatmeal, blueberries, soymilk 2. Whole grain pasta, marinara sauce, baked fish, coffee 4. Lentil soup, vegetable medley, fruit salad, water 3. Correct: This is not a good choose for this client. Meat is high fat. French bread with butter is low fiber and high fat. 1. Incorrect: This is a good meal choose when on a low fat, high fiber diet. Blueberries are high in fiber and all are low fat. 2. Incorrect: This is a good low fat, high fiber meal choose. Whole grain pasta is high in fiber and low in fat. Fish and marinara sauce are low in fat. 4. Incorrect: These are low fat, high fiber items to consume. Question: The client had a thoracentesis with removal of 2500 mL of fluid from the chest cavity. What is the priority nursing assessment for this client? You answered this question Correctly 2. Pain 3. O2 sat 4. Signs of infection 1. Correct: That’s right. Should be watching the vital signs for shock, tachycardia, and hypotension because a lot of fluid has just been removed from the body. 2. Incorrect: Not priority; remember, pain never killed anyone. 3. Incorrect: We will watch but isn’t highest priority. 4. Incorrect: Monitoring vital signs would show signs of infection. Question: Upon receiving a diagnosis of Stage 4 lung cancer, an elderly client expresses regret for having chosen to smoke. Which response by the nurse would best help the client cope at this time? You answered this question Correctly 1. "You are lucky to have lived a very long life." 2. "We have younger clients in worse shape than you." 3. "The doctor will make sure to treat any pain." 4. Correct: The nurse responds with an open-ended statement that reflects back what the client has stated. This allows the client to continue expressing concerns and feelings about the diagnosis or past choices. At this time, encouraging the client to verbalize is the best choice to help with coping. 1. Incorrect: This is a closed-ended statement that diminishes the client's feelings about the diagnosis or possible personal choices that may have led to this situation. The response infers that the client should be grateful for the life lived and belittles the client’s response. 2. Incorrect: Such a non-therapeutic statement does not address the client's feelings and in fact devalues the client by comparing this situation to that of other clients. The nurse should focus on helping the client to cope at this time by encouraging the expression of feelings. 3. Incorrect: Rather than allowing the client to verbalize concerns or emotions, the nurse has responded with a close-ended statement that addresses a topic not initiated by the client's comments. This option does nothing to help the client cope, but rather may instill fear by referring to pain that may or may not occur. Question: The nurse is caring for a client diagnosed with type 2 diabetes who was brought to the emergency department in an unresponsive state. A diagnosis of hyperglycemic hyperosmolar nonketotic syndrome (HHNS) is made. The nurse prepares for the administration of which initial therapy? You answered this question Correctly 1. Oxygen by nasal cannula 2. Long-acting IV insulin 4. IV dextran 3. Correct: Clients in HHNS diurese due to a high glucose load in the vascular space. The client becomes severely volume depleted and is at risk for developing shock. Therapy is focused on combating shock. 1. Incorrect: Oxygen by nasal cannula is not the priority for this client. Don’t pick oxygen as a priority every time. Oxygen does not fix the problem. The problem is shock. 2. Incorrect: The client will be given short-acting insulin. 4. Incorrect: Dextran is contraindicated as this will increase blood sugar even more. Question: What should a community health nurse include when planning a presentation on prevention and early detection of colon cancer? You answered this question Incorrectly 3. Regular screening should begin at age 30. 5. Flexible esophagogastroduodenoscopy every 5 years. 1., 2., & 4. Correct: A diet high in vegetables, fruits, and whole grains has been linked with a decreased risk of colorectal cancer; whereas, a diet high in red meats, processed meats, and cooking meats at very high temperature (frying, broiling or grilling) creates chemicals that may increase the risk for colorectal cancer. There is a greater risk of developing colorectal cancer in individuals who live a sedentary life style. The guaiac- based fecal occult blood test detects blood in the stool through a chemical reaction. This test is done yearly. 3. Incorrect: If there are no identified risk factors (other than age), regular screening should begin at age 50. 5. Incorrect: Flexible sigmoidoscopy looks at the rectum and colon to detect polyps and colon cancer. For people who have none of the risks described earlier, digital rectal examination and testing of the stool for hidden blood are recommended annually beginning at age 40. Flexible sigmoidoscopy is recommended every 5 years at age 50 or older. A double contrast barium enema every 5 to 10 years and colonoscopy every 10 years are acceptable alternatives. Question: The nurse is caring for a 5-year old child brought to the Emergency Department by the parents for pain and swelling in the left arm. An x-ray of the arm confirmed a fracture. The parents give conflicting stories about the accident. What action by the nurse is most appropriate? You answered this question Incorrectly 1. Prepare the child for casting of the arm. 2. Ask the primary healthcare provider to order bone series film. 4. Obtain a history as to how the accident happened. 3. Correct: All states have laws that mandate reporting of child maltreatment. Usually the social service department handles these types of referrals. 1. Incorrect: Casting may be needed, but the most appropriate action is identifying safety issues and possible child abuse. 2. Incorrect: X-rays would be the primary healthcare provider’s decision. 4. Incorrect: There is nothing wrong with this answer, but you only have one chance to tell the NCLEX people you know the most important thing to do is to consult social services. This is a safety issue. Question: What should the nurse include when providing teaching to a female client prescribed doxycycline for the treatment of acne? You answered this question Correctly 1. Take this medication with food to maximize absorption. 5. Iron and calcium supplements can be taken with this medication. 2. , 3., & 4. Correct: Doxycycline is a tetracycline antibiotic. Doxycycline can make birth control pills less effective. A non-hormone method of birth control (such as a condom, diaphragm, spermicide) should be used to prevent pregnancy while using doxycycline. Avoid exposure to sunlight or tanning beds. Doxycycline can make you sunburn more easily. Wear protective clothing and use sunscreen (SPF 30 or higher) when outdoors. Take doxycycline with a full glass of water. Drink plenty of liquids while taking this medicine. 1. Incorrect: Take on an empty stomach to maximize absorption, although may not be tolerated unless administered with food. 5. Incorrect: Do not take iron supplements, multivitamins, calcium supplements, antacids, or laxatives within 2 hours before or after taking doxycycline. Absorption will be altered. For instance, iron can bind to doxycycline in the gastrointestinal tract, which may prevent their absorption into the bloodstream and possibly reduce their effectiveness. To avoid or minimize the interaction, iron containing medications and doxycycline should preferably be taken at least three hours apart in most cases. Question: The nurse is assessing pain after surgery in a 3 year old client with a known developmental delay. Which pain scale should the nurse use to assess this client’s pain level? You answered this question Incorrectly 1. CRIES scale 2. Numeric scale 4. FACES scale 3. Correct: The FLACC scale can be used for pediatric clients from age 2 months to 7 years and is appropriate when clients cannot communicate their pain. 1. Incorrect: The CRIES scale is used with neonates and infants. 2. Incorrect: Not age-appropriate; used for children ages 5 and up. 4. Incorrect: Not age-appropriate. The FACES scale is indicated for children ages 3 years and up. When using the FACES scale, the child must be able to understand the difference between pain and being sad. Because this child is only 3 years old (the bottom age for use of the FACES scale), and because the client has a developmental delay, the FLACC scale is a better choice as it is based on nursing observations. Question: Which symptoms would the nurse be likely to observe in the client who overdosed on diazepam? You answered this question Correctly 3. Hyperthermia 5. Hyperreflexia 6. Psychosis 1., 2., & 4. Correct: Benzodiazepines are central nervous system (CNS) depressants. Diazepam is a benzodiazepine. They will slow respirations (bradypnea) and the heart rate (bradycardia). Somnolence (extreme, prolonged drowsiness) would be seen. 3. Incorrect: Benzodiazepines would not cause hyperthermia. 5. Incorrect: Benzodiazepines would diminish reflexes since it is a CNS depressant. 6. Incorrect: Psychosis is not a common symptom with CNS depression. Question: A home health nurse is assessing the home environment for safety issues concerning ambulation. Which finding would require the nurse to counsel the client and family? You answered this question Correctly 2. Grab bar in bath tub 3. Nonskid strips on outside steps 1. , 4., & 5. Correct: Rooms and hallways should have adequate lighting so client can see while ambulating and see any objects which may be in the way. Throw rugs (rugs that are not secured) can slide and cause a fall. Slippery floors will contribute to falls. 2. Incorrect: Adequate supports such as railings and grab bars can help prevent falls. 3. Incorrect: Having nonskid strips on outside steps and inside stairs help prevent falls. Question: Which instructions should the nurse give the unlicensed assistive personnel (UAP) about care needed to reduce the risk of infection when a client has an indwelling catheter? You answered this question Correctly 2. Instruct the UAP to disconnect the catheter from the bag when measuring output. 1. , 3., 4. & 5. Correct: Tubing that becomes obstructed cannot allow adequate urine flow. The urine flow occurs by gravity. Adequate handwashing before providing care is one defense against infection. Tension on the tubing may cause irritation and subsequent infection. The bag should be below the level of the bladder so that urine flows appropriately. 2. Incorrect: A closed drainage system should be maintained to prevent entry of microorganisms. This would be incorrect and potentially cause harm to the client. Question: A nurse is caring for a client hospitalized with Guillain-Barre syndrome. Which is the most important nursing measure to include in the nursing care plan for this client? You answered this question Correctly 2. Insertion of indwelling urinary catheter 3. Nasogastric suctioning 4. Frequent assessments of level of consciousness 1. Correct: Guillain-Barre syndrome is an acquired inflammatory disease that results in demyelinization of the peripheral nerves. It is usually ascending in nature and can lead to respiratory paresis or paralysis. 2. Incorrect: Insertion of an indwelling urinary catheter may in fact be necessary but does not prioritize higher than support of ventilation. 3. Incorrect: Nasogastric suctioning is not a need identified with Guillain-Barre syndrome. Guillain-Barre does not affect the LOC. 4. Incorrect: The client’s cognitive function remains intact, and there is no data in the stem of the question that indicates otherwise; therefore, ventilation is the priority. Question: A client, admitted to the medical unit with persistent vomiting, reports weakness and leg cramps. The spouse states that the client is irritable. The primary healthcare provider has prescribed lab work and blood gases. Based on this assessment, the nurse anticipates which acid/base imbalance? You answered this question Correctly 1. Respiratory acidosis 2. Respiratory alkalosis 3. Metabolic acidosis 4. Correct: Symptoms of alkalosis are often due to associated potassium loss and may include irritability, weakness, and cramping. Excessive vomiting eliminates gastric acid and potassium, leading to metabolic alkalosis. 1. Incorrect: Not respiratory related acid base imbalance. 2. Incorrect: Not respiratory related acid base imbalance. 3. Incorrect: Not acidosis. There is loss of gastric acid and K with persistent vomiting. Question: The women's health charge nurse is making assignments for the next shift. The unit is short one staff member and will receive a nurse from the medical surgical unit. Which clients should the charge nurse assign to the medical-surgical nurse? You answered this question Incorrectly 3. Vaginal delivery with fetal demise 4. 32 week gestation with lymphoma 5. Post-partal with HELLP syndrome 1., & 2. Correct: Though the clients are females, their postoperative care has similarities to the standard postoperative clients. 3. Incorrect: This client needs specialized care. This postpartum client had a vaginal delivery. A nurse who has experience caring for a client who delivered a stillborn should be assigned to this client. 4. Incorrect: This client needs specialized care. The client is at 32 weeks gestation. A nurse with obstetrical experience, should be assigned to this client. 5. Incorrect: No, the monitoring is too specific for the medical-surgical nurse. Hemolysis Elevated Liver enzymes Low Platelet count (HELLP) syndrome is a form of preeclampsia with severe liver damage. The medical-surgical nurse should not be assigned to this client. Question: A client is hospitalized for chronic renal failure. The nurse will need to notify the primary healthcare provider concerning which findings? You answered this question Correctly 1. Sodium 135 mEq/L 3. BP 100/70 4. No weight loss 2. , 5. Correct: Normal K 3.5-5.0 mEq/L; Normal ionized serum Ca 4.5-5.5 mg/dL. The abnormal lab results need to be reported. 1. Incorrect: Normal sodium 135-145 mEq/L. 3. Incorrect: Hypertension is a potential complication of chronic renal failure. 4. Incorrect: Desired outcome: client exhibits no rapid increases or decreases in weight. Question: A staff nurse decides to go to lunch with a friend instead of meeting with a study group for a certification exam. The staff nurse informs the clinical specialist, "Studying more will not do any good anyway." What defense mechanism does the clinical specialist understand that the staff nurse is exhibiting? You answered this question Correctly 2. Denial 3. Regression 4. Reaction formation 1. Correct: Rationalization is the mind’s way of justifying behavior by offering an explanation other than a truthful response. This is often used to avoid embarrassment. 2. Incorrect: Denial is the unconscious refusal to acknowledge painful realities, feelings, or experiences. It offers a temporary escape from an unpleasant event. 3. Incorrect: Regression is a reversion to immature patterns of behavior. 4. Incorrect: Reaction formation is behaving in a way that is exactly opposite of one’s true feelings. Question: A client, who is having difficulty falling asleep, asks the nurse for a sleeping aid. What is the first action the nurse should provide to the client? You answered this question Correctly 1. Assist client to take a cool bath. 3. Administer prescribed triazolam. 4. Give client a crossword puzzle to work. 2. Correct: Research indicates that back massage can enhance client comfort, relaxation, and sleep. This is the least invasive option and should be done first. 1. Incorrect: A cool bath would wake a client, whereas, a warm bath would increase relaxation. 3. Incorrect: Triazolam is a short acting benzodiazepine. Do not go to the sleeping pill first. 4. Incorrect: Distraction is a good strategy for drawing a client’s attention away from pain but may increase thinking, thus keeping the client awake. Question: A small community has experienced a severe tornado that hit a shopping mall and caused extreme damage and suspected mass casualties and injuries. Which intervention takes priority? You answered this question Incorrectly 1. Triage victims and tag according to injury. 2. Assess the immediate area for electrical wires on the ground and in vicinity of victims. 4. Begin attending to injuries as they are encountered. 3. Correct: With mass casualties, community response teams are needed. 1. Incorrect: This would be the third step. 2. Incorrect: This would be the second step so that further injuries are not encountered. 4. Incorrect: Triage must occur before treatment of anyone so that an accurate assessment of level of injuries can be made. With mass casualties, a color tag system is usually implemented. Question: The nurse’s goal is to reduce the risk of flu and its complications by offering a class at the local high school. Which groups of people should be included in the nurse's teaching plan as needing the flu shot? You answered this question Correctly 1. Babies less than 6 months old 2. , 3., 4. & 5. Correct: All people greater than 6 months of age should get a flu shot, unless allergic to eggs, or if there has been an adverse reaction in the past. Pregnant women should receive the flu shot. Parents of young children may be exposed to the flu and should get the vaccine. People with a chronic illness are more susceptible to flu and its complications. 1. Incorrect: Flu vaccine has not been approved for babies less than 6 months old. Question: The clinical specialist is teaching a group of new staff nurses about therapeutic communication. Which statement by one of the staff nurses indicates to the clinical specialist that further teaching is needed? You answered this question Correctly 1. Effective communication involves feedback to let the sender know that the message was understood by the receiver. 2. An effective message should be clear and complete. 4. I must listen with a "third ear" to be aware of what the client is not saying. 3. Correct: Gestures are a type of nonverbal communication which can provide assistance in communicating therapeutically with a client. Other forms of nonverbal communication include facial expression, touch, mannerisms, posture, position, and personal space. 1. Incorrect: This is a correct statement regarding therapeutic communication. 2. Incorrect: This is a correct statement regarding therapeutic communication. 4. Incorrect: This is a correct statement regarding therapeutic communication. The third ear listens for what the client is not saying or picks up on hints as to the real message. Question: Which foods should the nurse teach a client to avoid when prescribed a diet limiting purine rich foods? You answered this question Correctly 1. Peanut butter 2. Potatoes 3. Apples 4. , & 5. Correct: Meats such as liver, bacon, veal, and venison are high in purine and should be avoided. Seafood such as sardines, mussels, codfish, scallops, trout, and haddock are high in purine and should be avoided. 1. , 2., & 3. Incorrect: The client can enjoy peanut butter, potatoes, fruits and fruit juices, vegetables. Question: The nurse is working with the parents of a preschooler to help promote healthy sleep patterns of approximately 8–12 hours per night. Which intervention should assist the parents to achieve adequate sleep for their preschooler? You answered this question Correctly 1. Offer a time of exercise prior to bedtime. 2. Follow a bedtime routine at least three or four nights per week. 4. Do not encourage your preschooler to take a toy to bed. 3. Correct: Rituals help the preschooler to feel secure. Quiet time to read, tell stories, and say prayers prepares the child for sleep. 1. Incorrect: Stimulation of activity before bedtime impedes sleep. 2. Incorrect: The routine should be maintained each night if at all possible. Only through routine does the child feel secure in preparation for bedtime. 4. Incorrect: A special toy helps the child to feel secure and adds to the nighttime routine. Question: The nurse is working with the parents of a preschooler to help promote healthy sleep patterns of approximately 8–12 hours per night. Which intervention should assist the parents to achieve adequate sleep for their preschooler? You answered this question Correctly 1. Offer a time of exercise prior to bedtime. 2. Follow a bedtime routine at least three or four nights per week. 4. Do not encourage your preschooler to take a toy to bed. 3. Correct: Rituals help the preschooler to feel secure. Quiet time to read, tell stories, and say prayers prepares the child for sleep. 1. Incorrect: Stimulation of activity before bedtime impedes sleep. 2. Incorrect: The routine should be maintained each night if at all possible. Only through routine does the child feel secure in preparation for bedtime. 4. Incorrect: A special toy helps the child to feel secure and adds to the nighttime routine. Question: The nurse is teaching a client who has been prescribed daily glucocorticoids for the treatment of Addison’s disease. What teaching points should the nurse emphasize? You answered this question Incorrectly 1. Be aware of the development of hypoglycemia. 2. Test the urine for albumin or other proteins. 3. Take the medication 30 minutes prior to bedtime. 4. Correct: Glucocorticoids should not be abruptly discontinued due to the risk of adrenal suppression with prolonged use. Doses should be tapered before being discontinued. 1. Incorrect: Increased blood sugar is an adverse effect associated with glucocorticoid therapy, not hypoglycemia. 2. Incorrect: Protein in the urine is not associated with glucocorticoid therapy. 3. Incorrect: Insomnia is an adverse effect associated with glucocorticoid therapy. Daily dosing of glucocorticoids should be done in the morning to decrease this effect. Question: Following a total hip replacement, the nurse provides discharge teaching to the client. The nurse knows that teaching was effective when the client states which activities are safe to perform? You answered this question Correctly 2. Crossing the legs 5. Tying shoes 1. , 3., & 4. Correct: The client should use an abduction pillow to keep hip in proper alignment and prevent hip dislocation. A toilet extender keeps the hip in proper alignment and prevents hip dislocation. Showering rather than sitting in a tub will prevent flexion of the hip. 2. Incorrect: Crossing the leg can pop the hip out of place and prevent total healing and success with the replacement. 5. Incorrect: To tie shoes, the client has to bend over which can pop the hip out of place. The client would need to have shoes that do not require tying or have someone do it for them. Question: Which assignment would be most appropriate for the nurse to delegate to an unlicensed assistive personnel (UAP)? You answered this question Correctly 1. Obtaining a sterile urine specimen from an indwelling catheter. 2. Inserting an in-and-out catheter on a client postpartum. 4. Removing an indwelling catheter on a client postpartum. 3. , & 5. Correct: Taking vital signs is within the scope of practice for the UAP, but the nurse is responsible for evaluating the vital signs. Performing perineal care is within the scope of practice for the UAP. The nurse is responsible for assessing the episiotomy and confirming that perineal care is done properly. 1. Incorrect: Invasive procedures are not appropriate tasks for UAP (obtaining sterile specimen from indwelling catheter). 2. Incorrect: Invasive procedures are not appropriate tasks for UAP (inserting catheter). 4. Incorrect: Invasive procedures are not appropriate tasks for UAP (removing indwelling catheter). Question: The nurse is caring for a client taking spironolactone. Which dietary change should the nurse teach the client to make when starting treatment with this medication? You answered this question Correctly 1. Eat extra helpings of bananas. 2. Increase intake of water. 4. Increase intake of green leafy vegetables. 3. Correct: Spironolactone is a potassium sparing diuretic. Salt substitutes have potassium instead of sodium and should be avoided. 1. Incorrect: Bananas have potassium and should be avoided to prevent hyperkalemia and life-threatening arrhythmias. 2. Incorrect: Water intake does not affect the use of spironolactone and is not the best answer to the question. 4. Incorrect: Green leafy vegetables contain vitamin K and are not contraindicated. This is a distractor to the answer that could cause the most harm to the client if not chosen. Question: A nurse who has never had varicella has been exposed to a client diagnosed with herpes zoster. What actions should the nurse take? You answered this question Incorrectly 2. Continue to care for client as varicella and herpes zoster are not related. 3. Go to the lab to have a Tzanck smear performed. 4. Obtain herpes zoster vaccine for protection from this exposure. 1. , & 5. Correct: Notify the person responsible for infection control to get post-exposure treatment initiated within a timely manner. For persons who are susceptible, the varicella-zoster immune globulin should be given within 96 hours of exposure. The infection of herpes zoster is contagious until the crusts have dried and fallen off the skin. 2. Incorrect: Varicella is chickenpox and herpes zoster is shingles. Both are closely related. Exposure to herpes zoster by someone who has not had varicella places the person at risk for developing herpes zoster. 3. Incorrect: A Tzanck test consists of examining tissue from the lower surface of a lesion in a vesicular condition to determine cell type. The Tzanck test is not associated with immunity from the varicella-zoster virus. 4. Incorrect: The vaccine will not prevent the nurse from developing shingles from this exposure. The nurse needs immune globulin for immediate protection. [Show More]

Last updated: 1 year ago

Preview 1 out of 74 pages

Also available in bundle (1)

RNSG 2331 - Exam 1-3-capstone-1 PACKAGE DEAL (100% correct answers with Rationales)

RNSG 2331 - Exam 1-3-capstone-1 PACKAGE DEAL (100% correct answers with Rationales)

By A+ Solutions 1 year ago

$14.5

4  

Reviews( 0 )

$14.00

Add to cart

Instant download

Can't find what you want? Try our AI powered Search

OR

GET ASSIGNMENT HELP
65
0

Document information


Connected school, study & course


About the document


Uploaded On

Oct 18, 2022

Number of pages

74

Written in

Seller


seller-icon
A+ Solutions

Member since 3 years

164 Documents Sold


Additional information

This document has been written for:

Uploaded

Oct 18, 2022

Downloads

 0

Views

 65

Document Keyword Tags

Recommended For You


$14.00
What is Browsegrades

In Browsegrades, a student can earn by offering help to other student. Students can help other students with materials by upploading their notes and earn money.

We are here to help

We're available through e-mail, Twitter, Facebook, and live chat.
 FAQ
 Questions? Leave a message!

Follow us on
 Twitter

Copyright © Browsegrades · High quality services·